Mathematical and Physical Journal
for High Schools
Issued by the MATFUND Foundation
Already signed up?
New to KöMaL?

Problem B. 4905. (November 2017)

B. 4905. Let \(\displaystyle a_1 \ge a_2 \ge a_3 \ge \cdots \ge a_{2n-1} \ge a_{2n} \ge 0\), and \(\displaystyle \sum_{i=1}^{2n} a_i = 1\). Prove that

\(\displaystyle a_1 a_2 + 3 a_3 a_4 + 5 a_5 a_6 + \ldots + (2n-1) a_{2n-1} a_{2n} \le \frac{1}{4}. \)

When will the equality hold?

(4 pont)

Deadline expired on December 11, 2017.


Sorry, the solution is available only in Hungarian. Google translation

1. megoldás. Mivel \(\displaystyle 0\leq (x-y)^2\), ahol egyenlőség csak \(\displaystyle x=y\) esetén teljesül, ezért

\(\displaystyle xy\leq\frac{4xy+(x-y)^2}{4}= \frac{(x+y)^2}{4},\)

ahol egyenlőség szintén csak \(\displaystyle x=y\) esetén teljesül.

Ezt használva kapjuk, hogy bármely \(\displaystyle 1\leq k\leq n\) mellett \(\displaystyle a_{2k-1}a_{2k}\leq \frac{(a_{2k-1}+a_{2k})^2}{4}\), ahol egyenlőség csak \(\displaystyle a_{2k-1}=a_{2k}\) esetén teljesül. Ha tehát minden \(\displaystyle k\)-ra \(\displaystyle a_{2k-1}\)-et és \(\displaystyle a_{2k}\)-t kicseréljük \(\displaystyle \frac{a_{2k-1}+a_{2k}}{2}\)-re, akkor \(\displaystyle a_1a_2+3a_3a_4+\dots+(2n-1)a_{2n-1}a_{2n}\) értéke biztosan nem csökken, és csak akkor nem nő, ha az \(\displaystyle a_1=a_2,a_3=a_4,\dots,a_{2n-1}=a_{2n}\) egyenlőségek eredetileg is teljesültek.

Meg fogjuk mutatni, hogy ekkor

\(\displaystyle a_1a_2+3a_3a_4+\dots+(2n-1)a_{2n-1}a_{2n}=a_1^2+3a_3^2+\dots+(2n-1)a_{2n-1}^2\leq \frac14\)

biztosan teljesül. Ez egyben az eredeti állítás speciális esete, hiszen az \(\displaystyle a_1\geq a_2\geq \dots a_{2n}\geq 0\) és \(\displaystyle \sum\limits_{i=1}^{2n} a_i=1\) feltételek továbbra is teljesülnek.

Felhasználva, hogy \(\displaystyle \sum\limits_{i=1}^{2n} a_i=1\), a bizonyítandó egyenlőtlenség:

\(\displaystyle a_1^2+3a_3^2+\dots+(2n-1)a_{2n-1}^2\leq \frac{1}{4}\left(\sum\limits_{i=1}^{2n} a_i\right)^2,\)

vagyis

\(\displaystyle a_1^2+3a_3^2+\dots+(2n-1)a_{2n-1}^2\leq \left(\sum\limits_{i=1}^{n} a_{2i-1}\right)^2.\)

További ekvivalens átalakításokat végrehajtva:

\(\displaystyle \sum\limits_{i=2}^n 2(i-1) a_{2i-1}^2\leq \sum\limits_{1\leq j<i\leq n} 2a_{2j-1}a_{2i-1},\)

\(\displaystyle \sum\limits_{i=2}^n 2(i-1) a_{2i-1}^2\leq \sum\limits_{i=2}^n \left( 2a_{2i-1}\sum\limits_{1\leq j<i}a_{2j-1} \right).\)

Mivel \(\displaystyle \sum\limits_{1\leq j<i}a_{2j-1}\geq \sum\limits_{1\leq j<i}a_{2i-1} \geq (i-1)a_{2i-1}\), ezért ez az egyenlőtlenség biztosan teljesül.

Az egyenlőség teljesüléséhez minden \(\displaystyle 2\leq i\leq n\) esetén teljesülnie kell vagy annak, hogy \(\displaystyle a_1=a_3=\dots=a_{2i-1}\), vagy annak, hogy \(\displaystyle a_{2i-1}=0\). Ha ez az \(\displaystyle n-1\) feltétel mind teljesül (valamint \(\displaystyle a_{2k-1}=a_{2k}\) minden \(\displaystyle 1\leq k\leq n\)-re), akkor valóban fennáll az egyenlőség.

Az egyenlőség tehát a következő esetekben áll fenn: valamely \(\displaystyle 1\leq k\leq n\) mellett \(\displaystyle a_1=a_2=\dots=2_{2k-1}=a_{2k}=\frac{1}{2k}\), és \(\displaystyle a_{2k+1}=a_{2k}=\dots=a_{2n}=0\).

2. megoldás (vázlat). Azt bizonyítjuk be, hogy \(\displaystyle 1\le k\le n\) esetén

\(\displaystyle a_1 a_2 + 3 a_3 a_4 + 5 a_5 a_6 + \ldots + (2k-1) a_{2k-1} a_{2k} \le \frac{(a_1+a_2+\ldots+a_{2k})^2}{4}. \tag1 \)

A \(\displaystyle k=n\) esetben ez éppen a bizonyítandó állítás.

A \(\displaystyle k\) szerinti indukcióval bizonyítunk. A \(\displaystyle k=1\) esetben az állítás

\(\displaystyle a_1 a_2 \le \frac{(a_1+a_2)^2}{4}, \)

ami \(\displaystyle 4\)-gyel szorozva és egy oldalra rendezve

\(\displaystyle (a_1-a_2)^2 \ge 0; \)

egyenlőség csak \(\displaystyle a_1=a_2\) esetén áll fenn.

Az indukciós lépéshez elég azt igazolnunk, hogy

\(\displaystyle (2k-1)a_{2k-1}a_{2k} \le \frac{(a_1+a_2+\ldots+a_{2k})^2}{4} -\frac{(a_1+a_2+\ldots+a_{2k-2})^2}{4} \)

azaz

\(\displaystyle (2k-1)a_{2k-1}a_{2k} \le \frac{(a_1+a_2+\ldots+a_{2k-2})(a_{2k-1}+a_{2k})}{2} + \frac{(a_{2k-1}+a_{2k})^2}{4}. \tag2 \)

A jobboldalon

\(\displaystyle \frac{(a_1+a_2+\ldots+a_{2k-2})(a_{2k-1}+a_{2k})}{2} \ge \frac{(2k-2)a_{2k-1} \cdot 2a_{2k}}{2}; \tag3 \)

egyenlőség kétféleképpen állhat fenn: ha mindkét tényezőnél egynlőség van, vagyis \(\displaystyle a_1=a_2=\ldots=a_{2k-2}=a_{2k-1}\), illetve \(\displaystyle a_{2k-1}=a_{2k}\), vagy pedig a második tényező \(\displaystyle 0\), azaz \(\displaystyle a_{2k-1}+a_{2k}=0\).

A (2) utolsó tagjában

\(\displaystyle \frac{(a_{2k-1}+a_{2k})^2}{4} \ge a_{2k-1} a_{2k} \tag4 \)

ekvivalens azzal, hogy \(\displaystyle (a_{2k-1}-a_{2k})^2\ge0\), egyenlőség akor van, ha \(\displaystyle a_{2k-1}=a_{2k}\).

A (2) és (3) becslések együtt igazolják (2), így az indukció működik.

Az állításban akkor áll fenn egyenlőség, ha mindegyik induciós lépésnél egyenlőség áll fenn; a megoldásból leolvasható, hogy vagy \(\displaystyle a_1=\ldots=a_{2n}=\frac1{2n}\), vagy pedig valamelyik \(\displaystyle 1\le k<n\) esetében \(\displaystyle a_1=\ldots=a_{2k}=\frac1{2k}\) és \(\displaystyle a_{2k+1}=\ldots=a_{2n}=0\).


Statistics:

69 students sent a solution.
4 points:Bukva Dávid, Fülöp Anna Tácia, Gáspár Attila, Janzer Orsolya Lili, Kerekes Anna, Kovács 129 Tamás, Kupás Vendel Péter, Nagy Nándor, Terjék András József, Weisz Máté.
3 points:Ajtai Boglárka, Beke Csongor, Csiszár Zoltán, Deák Bence, Fraknói Ádám, Füredi Erik Benjámin, Geretovszky Anna, Győrffi Ádám György, Jánosik Máté, Jedlovszky Pál, Kitschner Bernadett, Kocsis Anett, Laki Anna, Lukács Lilla Réka, Márton Dénes, Mikulás Zsófia, Molnár Bálint, Olosz Adél, Richlik Róbert, Sulán Ádám, Tiderenczl Dániel, Tóth 827 Balázs, Tubak Dániel, Williams Hajna.
2 points:8 students.
1 point:21 students.
0 point:6 students.

Problems in Mathematics of KöMaL, November 2017